LSAT and Law School Admissions Forum

Get expert LSAT preparation and law school admissions advice from PowerScore Test Preparation.

 Administrator
PowerScore Staff
  • PowerScore Staff
  • Posts: 8916
  • Joined: Feb 02, 2011
|
#40636
Complete Question Explanation
(The complete setup for this game can be found here: lsat/viewtopic.php?t=4425)

The correct answer choice is (B)

If K is the sole paralegal assigned to one of the cases, and from the second rule there must be another case with just F or G, then the 3-1-1 distribution must be in effect, in the following alignment:

PT69_Game_#4_#23_diagram 1.png
Accordingly, because H is assigned to Sicoli, the group of three paralegals must be assigned to Sicoli. Accounting for the first rule, that leaves only two possible scenarios:

K assigned to Raimes:

PT69_Game_#4_#23_diagram 2.png
K assigned to Thompson, which would force F to be assigned to Raimes, per the first rule:

PT69_Game_#4_#23_diagram 3.png
Consequently, only F or K can ever be assigned to Raimes, and answer choice (B) is correct.
You do not have the required permissions to view the files attached to this post.
 tnafisi
  • Posts: 2
  • Joined: Nov 18, 2013
|
#12573
I do not understand what this question is asking for and how it came up with answer B. Frank and Kevin. I do not understand what the answer means either. I thought Gina could also be a sole paralegal for the raimes case. Please break down and explain this question for me. thank you.
User avatar
 Dave Killoran
PowerScore Staff
  • PowerScore Staff
  • Posts: 5853
  • Joined: Mar 25, 2011
|
#12611
Hi T,

The questions wants the complete list of every paralegal that could be assigned to Raimes' case, under any scenario where K is the sole paralegal assigned to one of the cases. Thus if one variable could be assigned to Raimes in one scenario, and another paralegal could be assigned to Raimes in a different scenario, both of those paralegals would need to appear in the correct answer. This "complete list of everyone who could ever appear" is a standard variation on the classic List questions you see that typically start games.

To accord with the condition in the question stem, a 3-1-1 distribution is in effect, with one of the 1s being K, and ultimately--due to the other rules--with the group of three assigned to Sicoli. Only two basic templates result from these assignments, one where F is assigned to Raimes, and one where K assigned to Raimes. In each case, they are the only paralegal assigned, and consequently, only F or K can ever be assigned to Raimes, and answer choice (B) is correct (I'd draw these out, but the graphical limitations of the Forum make that extremely hard to do).

You ask about G, but G cannot be assigned to Raimes as that would force F to be the lone paralegal assigned to Thompson, thereby violating the second rule.

That cover the basic pieces of this question, and should get you started. If you have further questions about it, please let me know. Thanks!

Get the most out of your LSAT Prep Plus subscription.

Analyze and track your performance with our Testing and Analytics Package.